Letter to the editor: The Planning Department budget increased from $100,000 in 2001 to $524,000 for this year. Howe...

HolyBoy on September 28, 2015

Number 3

Doesn't E weaken the conclusion by saying they have more duties now? Wouldn't that justify the budget increase ?

Replies
Create a free account to read and take part in forum discussions.

Already have an account? log in

Naz on September 29, 2015

The argument tries to explain why the conclusion of the editorial, that because the Planning Department budget increased from $100,000 in 2001 to $524,000 this year means that the department now spends five times as much money as it did in 2001 to perform the same duties, is not true.

So this means that the person writing the letter believes that there are more duties now than before, since it is indisputable that the department is now spending five times more money as it did in 2001, i.e. the only thing this person could dispute with the conclusion of the editorial is that it isn't for the same amount of duties.

To help support this claim, we need an answer choice that reiterates that there are more duties today than before. This is exactly what answer choice (E) does: a restructuring act, passed in 2003, broadened the duties of the Planning Department.

Hope that helps clear things up! Please let us know if you have any other questions.

Rossalyn-Delfino on September 6, 2018

I'm confused? It sounds like an assumption was made? Where did you get the restructuring act from?

Max-Youngquist on September 6, 2018

@rossalyn-Delfino the restructuring act is introduced as a new premise in (E), it is not part of the passage. I hope that helps!

Minerva on July 1, 2019

I understand why E is correct but could you explain why D is incorrect? Thanks!

Gabrielaurr on September 18, 2019

Please explain why D is incorrect